Difference between revisions of "1985 AJHSME Problems/Problem 24"

(Blanked the page)
(Tag: Blanking)
(Video Solution)
 
(3 intermediate revisions by 3 users not shown)
Line 1: Line 1:
 +
==Problem==
  
 +
In a magic triangle, each of the six [[whole number|whole numbers]] <math>10-15</math> is placed in one of the [[circle|circles]] so that the sum, <math>S</math>, of the three numbers on each side of the [[triangle]] is the same.  The largest possible value for <math>S</math> is
 +
 +
<asy>
 +
draw(circle((0,0),1));
 +
draw(dir(60)--6*dir(60));
 +
draw(circle(7*dir(60),1));
 +
draw(8*dir(60)--13*dir(60));
 +
draw(circle(14*dir(60),1));
 +
draw((1,0)--(6,0));
 +
draw(circle((7,0),1));
 +
draw((8,0)--(13,0));
 +
draw(circle((14,0),1));
 +
draw(circle((10.5,6.0621778264910705273460621952706),1));
 +
draw((13.5,0.86602540378443864676372317075294)--(11,5.1961524227066318805823390245176));
 +
draw((10,6.9282032302755091741097853660235)--(7.5,11.258330249197702407928401219788));
 +
</asy>
 +
 +
<math>\text{(A)}\ 36 \qquad \text{(B)}\ 37 \qquad \text{(C)}\ 38 \qquad \text{(D)}\ 39 \qquad \text{(E)}\ 40</math>
 +
 +
==Solution 2==
 +
 +
To make the sum the greatest, put the three largest numbers <math>(13,14</math> and <math>15)</math> in the corners. Then, balance the sides by putting the least integer <math>(10)</math> between the greatest sum <math>(14</math> and <math>15)</math>. Then put the next least integer <math>(11)</math> between the next greatest sum (<math>13 +15</math>). Fill in the last integer <math>(12)</math> and you can see that the sum of any three numbers on a side is (for example) <math>14 +10 + 15 = 39</math>
 +
<math>\boxed{\text{D}}</math>.
 +
-by goldenn
 +
 +
==See Also==
 +
 +
{{AJHSME box|year=1985|num-b=23|num-a=25}}
 +
[[Category:Introductory Number Theory Problems]]
 +
[[Category:Introductory Algebra Problems]]
 +
 +
 +
{{MAA Notice}}

Latest revision as of 10:05, 10 June 2024

Problem

In a magic triangle, each of the six whole numbers $10-15$ is placed in one of the circles so that the sum, $S$, of the three numbers on each side of the triangle is the same. The largest possible value for $S$ is

[asy] draw(circle((0,0),1)); draw(dir(60)--6*dir(60)); draw(circle(7*dir(60),1)); draw(8*dir(60)--13*dir(60)); draw(circle(14*dir(60),1)); draw((1,0)--(6,0)); draw(circle((7,0),1)); draw((8,0)--(13,0)); draw(circle((14,0),1)); draw(circle((10.5,6.0621778264910705273460621952706),1)); draw((13.5,0.86602540378443864676372317075294)--(11,5.1961524227066318805823390245176)); draw((10,6.9282032302755091741097853660235)--(7.5,11.258330249197702407928401219788)); [/asy]

$\text{(A)}\ 36 \qquad \text{(B)}\ 37 \qquad \text{(C)}\ 38 \qquad \text{(D)}\ 39 \qquad \text{(E)}\ 40$

Solution 2

To make the sum the greatest, put the three largest numbers $(13,14$ and $15)$ in the corners. Then, balance the sides by putting the least integer $(10)$ between the greatest sum $(14$ and $15)$. Then put the next least integer $(11)$ between the next greatest sum ($13 +15$). Fill in the last integer $(12)$ and you can see that the sum of any three numbers on a side is (for example) $14 +10 + 15 = 39$ $\boxed{\text{D}}$. -by goldenn

See Also

1985 AJHSME (ProblemsAnswer KeyResources)
Preceded by
Problem 23
Followed by
Problem 25
1 2 3 4 5 6 7 8 9 10 11 12 13 14 15 16 17 18 19 20 21 22 23 24 25
All AJHSME/AMC 8 Problems and Solutions


The problems on this page are copyrighted by the Mathematical Association of America's American Mathematics Competitions. AMC logo.png